LSAT and Law School Admissions Forum

Get expert LSAT preparation and law school admissions advice from PowerScore Test Preparation.

User avatar
 Dave Killoran
PowerScore Staff
  • PowerScore Staff
  • Posts: 5852
  • Joined: Mar 25, 2011
|
#85362
Complete Question Explanation
(The complete setup for this game can be found here: lsat/viewtopic.php?f=315&t=9170)

The correct answer choice is (E)

If J views the site on day 1, then as discussed during the setup the following diagram results:

G1-Q2-d1.png

Accordingly, answer choice (E) is correct. Note that M was our last variable placed in our discussion, and not surprisingly, it is the correct answer here.
You do not have the required permissions to view the files attached to this post.
 studyhelp20
  • Posts: 28
  • Joined: Dec 09, 2020
|
#82390
Hello Power Score Support Staff,

Could I please receive some help in explaining the correct answer to this problem? Also, could I have explanations for all of the incorrect answers. Especially why answer C is incorrect? Thanks for the help

Sincerely,
Brennan
User avatar
 KelseyWoods
PowerScore Staff
  • PowerScore Staff
  • Posts: 1079
  • Joined: Jun 26, 2013
|
#82416
Hi Brennan!

As I said in over on the board for the setup to this game, there is only one possible order that works if J is first. See if you can figure it out!

If J is first, H is second. Now what? What other rules come into play here? Think about the HG rule. Where would G have to go if H is second? What happens with the F rule? What options are left for L? Now think about what you know about K :longline: this one is tricky but remember to use your contrapositive for the K4 :arrow: L5 rule. Now there's only one slot left for M. See if you can figure it out for yourself first and then let us know if/where you get stuck!

Hope this helps!

Best,
Kelsey

Get the most out of your LSAT Prep Plus subscription.

Analyze and track your performance with our Testing and Analytics Package.